Math, asked by lambasheetal, 6 months ago

The value of (cot35^(@)-1)(cot32^(@)-1)(cot29^(@)-1)*(cot26^(@)-1) (cot23^(@)-1)*(cot22^(@)-1)*(cot19^(@)-1)*(cot16^(@)-1) *(cot13^(@)-1)*(cot10^(@)-1) is equal to​

Answers

Answered by MaheswariS
3

\textbf{Given:}

\mathsf{(cot35^\circ-1){\times}(cot32^\circ-1){\times}(cot29^\circ-1){\times}(cot26^\circ-1){\times}(cot23^\circ-1){\times}}

\mathsf{(cot22^\circ-1){\times}(cot19^\circ-1){\times}(cot16^\circ-1){\times}(cot13^\circ-1){\times}(cot10^\circ-1)}

\textbf{To find:}

\textsf{The value of}

\mathsf{(cot35^\circ-1){\times}(cot32^\circ-1){\times}(cot29^\circ-1){\times}(cot26^\circ-1){\times}(cot23^\circ-1){\times}}

\mathsf{(cot22^\circ-1){\times}(cot19^\circ-1){\times}(cot16^\circ-1){\times}(cot13^\circ-1){\times}(cot10^\circ-1)}

\textbf{Solution:}

\textbf{Formula used:}

\boxed{\begin{minipage}{6.5cm}$\\\\\mathsf{If\;A\;\&\;B\;are\;acute\;angles\;with\;A+B=45^\circ}\\\\\mathsf{then\;(cotA-1)(cotB-1)=2}\\$\end{minipage}}

\mathsf{Consider,}

\mathsf{(cot35^\circ-1){\times}(cot32^\circ-1){\times}(cot29^\circ-1){\times}(cot26^\circ-1){\times}(cot23^\circ-1){\times}}

\mathsf{(cot22^\circ-1){\times}(cot19^\circ-1){\times}(cot16^\circ-1){\times}(cot13^\circ-1){\times}(cot10^\circ-1)}

\mathsf{Rearranging}

\mathsf{=(cot35^\circ-1){\times}(cot10^\circ-1){\times}(cot32^\circ-1){\times}(cot13^\circ-1){\times}(cot29^\circ-1){\times}}

\mathsf{(cot16^\circ-1){\times}(cot26^\circ-1){\times}(cot19^\circ-1){\times}(cot23^\circ-1){\times}(cot22^\circ-1)}

\mathsf{=[(cot35^\circ-1){\times}(cot10^\circ-1)]{\times}[(cot32^\circ-1){\times}(cot13^\circ-1)]{\times}[(cot29^\circ-1){\times}}

\mathsf{(cot16^\circ-1)]{\times}[(cot26^\circ-1){\times}(cot19^\circ-1)]{\times}[(cot23^\circ-1){\times}(cot22^\circ-1)]}

\mathsf{Using\;the\given\;formula}

\mathsf{=2{\times}2{\times}2{\times}2{\times}2}

\mathsf{=32}

Find more:

PROVE THAT:

cos 12 cos 24 cos 36 cos 48 Cos 60 cos 72 cos 84 equal = 1 / 128​

https://brainly.in/question/9849513

Tan70°⋅tan65°−tan70°−tan65°=1 prove that

https://brainly.in/question/7910163

Similar questions